Internal Medicine Residency Thread

This forum made possible through the generous support of SDN members, donors, and sponsors. Thank you.
Status
Not open for further replies.

NotAProgDirector

Pastafarians Unite!
Staff member
Volunteer Staff
15+ Year Member
Joined
Oct 11, 2006
Messages
10,418
Reaction score
14,645
Hello!

I'm a Program Director at a university based, academic IM program in the New England area. I'm happy to field questions about IM residencies -- anything from applications, ERAS, NRMP, IM training, curricula, evaluation, fellowship applications, getting your first job after residency -- or anything else vaguely related to IM residency training.

I cannot evaluate your own personal application, answer what "your chances of getting an interview" are, or review your personal statement. Please try to keep your questions general, so that they can be as widely applicable as possible.

I'm also going to try an experiment -- I'm going to keep the second post in this thread as a Table of Contents / Index for the thread. We'll see if that works.

Members don't see this ad.
 
  • Like
Reactions: 2 users
Table of Contents: (WARNING: I gave up updating this long ago....)
  1. What do you recommend for Board Review?
  2. How did you get where you are, and where do you see Internal Medicine going?
  3. What do I do if I have been put on probation during medical school when applying to residency?
  4. What in an application for an IM residency carries the most weight?
  5. Do Program Directors routinely review FaceBook or MySpace pages of applicants?
  6. Multiple part question
    • When is the best time to apply to programs via ERAS?
    • Are Step 1 or Step 2 scores more important? What if my Step 1 score is not great?
    • Are Away Rotations a good or bad idea? How important are they?
  7. How do you transfer from PGY-1 to a new PGY-2 program, and how does that affect your chances of getting a competitve fellowship program?
  8. How does the process of specialization work, and what does "Board Certified" mean?
  9. Do you have any advice for new interns?
  10. How important are Step 1 vs Step 2 scores, especially if I am an IMG?
  11. Multiple part question
    • How important is the ranking of my medical school in residency selection?
    • How important are extracurricular activities (esp non medical ones) during medical school?
  12. How many Sub-I's should I do, and what should I do with the rest of my fourth year electives?
  13. Fellowships: Which are competitive, how do I apply, and what's important?
  14. Is it harder to get a fellowship if I apply after graduating from residency?
    (Unnumbered Q) How important is my "home" program vs a "top" program for fellowships?
  15. What does it mean by "Community program, University affiliated" in FREIDA?
  16. Is research necessary to get an IM residency?
  17. I'm an IMG with poor USMLE scores. What can I do to improve my application?
  18. Where should I apply? How do I know what programs are good? How do I know which are "realistic" for my application?
 
Last edited:
  • Like
Reactions: 1 user
Hi

What materials do you recommend for IM boards prep?

QUESTION #1

For questions, I recommend MKSAP. The questions seem the most realistic, although many of my residents report that the MKSAP questions seem harder than the actual ABIM questions. No data behind this assessment.

For a nice, comprehensive text in outline format, I recommend MedStudy to my residents.

Many other sources are out there -- best to shop around and see what works for you.
 
  • Like
Reactions: 1 user
Members don't see this ad :)
QUESTION #2

I was going to ask you some specific questions about the direction of internal medicine as a specialty, and also about fellowships vs. working as a hospitalist vs. private practice after residency.
:confused:
However, when I thought about it I thought that maybe it would be interesting for people who are considering IM as a career path to hear how you got where you are today.
:thumbup:
What considerations did you take in 1) doing this specialty 2) chosing to practice where and how you are practicing now? Also 3) what other opportunities were open to you? Did your career take you where you are now immdediately or did you experiment with different practice settings, academic vs. private etc.
:confused:
I understand this might seem personal but please dont feel like I am asking about any kind of personal information such as university or name of hospital.
:)
Also how are IM specialties changing since you were a resident and where do you see them going. Meaning are there too many cards/gi/renal/heme-onc specialists or are these fields going to be unfilled in the near future.

I personally notice a lot of senior residents graduating last year going into hospitalist jobs and then perhaps moving on to "less competetive" fellowships like critical care - I wonder if this is a real trend?

Please feel free to really add anything that you think might help student / residents who might not be in the "real world" yet to get a clear picture of IM and subspecialties as a career path.

:luck:

There's a lot of questions here. I'll try to be concise:

1. I fell in love with academic medicine. At my Med School, all the academic / teaching docs were specialists, so I assumed I would be one also. At my residency, the most impressive academics were the internists, and I realized I really liked everything, so academic GIM it was. I went straight into academic GIM -- it's harder to get into academics if you don't do so right out of residency/fellowship.

2. How is medicine changing? It's so hard to predict. First there are reports of too many specialists, then too few. I've given up trying to guess. I think GIM will split into pure inpatient (hospitalists) and outpatient (GIM) fields, which is happening now. Hospitalist Medicine will become a lifelong career path as more people go into it and the number of shifts / weekends covered decreases.
 
  • Like
Reactions: 1 users
QUESTION #3

Originally Posted by zoondel: This question is for faculty mentors, residency program directors, and administrators:

I'm a fourth year medical student at a good school, with excellent grades across all years (mostly Honors). Most of my clinical evaluations are very good, with many of them including stand-out comments about exceptional clinical performance and devotion to patient care.
I will be be applying for residency after I graduate medical school, as I'm taking a year to do research.
However, I was on academic probation for one year (from 3rd-4th) for deficiencies in professionalism that occurred in 2nd and 3rd year, mainly related to issues of attendance and personality conflicts with two attendings. (I have spent a great deal of time working on these issues in order to see where I went wrong and how to avoid it in the future, although I'm sure I've made a few more mistakes along the way.)

I've been informed that my Dean's letter and my transcript will note that I was on probation.

Here are my questions:
1. How will this affect my chances at getting into a good residency program? (note, I will not likely be applying to subspecialty programs)

2. How do I answer if/when I'm asked about this on interviews?

No one at my medical school has been able to guide me with this. I would greatly appreciate any advice or suggestions. Please feel free to be critical - realistic answers would be the most helpful as I continue to plan my career.

Thank you so much.

In evaluating a medical student with a "red flag" on their record, there are four major things I look at/for in the application:

  1. What exactly created the problem / probation? If it's failing a course, that's usually straightforward. If it's a professional issue as you describe, vague answers such as "personal problems", "health issues", "family issues" are all major concerns without some detail. Left to my imagination, I will assume the worst.
  2. Does this fit a pattern, or is it an anomaly? Is there a pattern of low board/shelf exam scores? Are there written comments from other rotations raising concerns about your performance?
  3. Have you demonstrated a pattern/track record of improvement? I need to be CERTAIN that this is completely resolved, and will not happen again. You want to demonstrate that, put back in a similar situation, the outcome would be satisfactory.
  4. Do you demonstrate insight into the problem and it's solution, and do you take full responsibility for your part? Every story has two sides, as I'm sure your's does. That being said, you need to be clear that you understand the problem, accept your role/responsibility, and have a solution to prevent further similar issues.

If I can answer all four of these questions/issues from the application itself, then a decision regarding an interview is made. So how do you do this?

  1. Dean's letter -- virtually all US medical schools will allow you to review your Dean's letter for accuracy before they are sent. You are not allowed to change the content of your Dean's letter, unless you feel it is truly inaccurate. Your first step is to see what details they put into the Dean's letter. Usually, they will be extremely vague, attempting to protect your privacy. If you think that a more open discussion of your problem would help your application, you should try to get your letter writer to explain the situation in the best possible light. Remember, it's in your Dean's office's best interest to get you into a residency program, so they should be at least somewhat helpful in this regard.
  2. The Personal Statement -- For an entire year on probation, I think you will absolutely need to use your PS to address it. Some would argue that you are better off not drawing attention to your deficiencies in your PS. I personally disagree, and in this case (with a big problem on your application) I don't think you have much of a choice. It is vital that you see your Dean's letter first, so that your PS and your Dean's letter descriptions match up. You will discuss in your PS all the issues listed above. (Note: I expect others may disagree with this advice, and suggest you get other opinions)
  3. Applications -- Because of this problem, you will need to apply more broadly and perhaps to lower tier programs. Hyper competitive programs (Rad/Derm/Urology/Plastics) are probably out of the question altogether. Gen Surg is also quite competitive. You really don't know how this is going to affect you. The plan is to get at least 10 interview offers -- if you go on 8 interviews you are very likely to match in IM, FP, Peds, etc. You will need to decide if you want to apply to a large number of programs up front, or if you want to start with 10 programs and then expand if you don't get enough interviews.
  4. Interviews -- You will be asked about this, for sure. If not, you need to bring it up yourself. In fact, you might want to think about doing that right at the beginning of the interview yourself, to get it out of the way ("Dr. XXX, before you start asking questions, I'd like to talk for a few minutes about the obvious concern about my academic probation on my record, what I have learned from this process, and how I have addressed this so that it will no longer be a problem")
  5. The confidential informant -- As a real ace-in-the-hole, see if you can find someone involved with your remediation who would be willing to talk to PD's about your issue off the record. They must be as unbiased as possible, know all the intimate details of the issue, and be willing to talk "off the record". You have to trust them completely. If you have such a person, during your visit make sure to tell the PD / interviewer that if they have any concerns, they can contact this person for more details.

Exactly how much this will affect your application depends on the details, and how it is addressed by you and your program. A study demonstrated that unprofessional behavior in med school predicts future board disciplinary actions. Many programs may simply see you as too much of a risk. PD's have all had residents who look great on paper, but have self destructive personalities that erode camaraderie and create discord in the residency. I would rather have an empty slot.
 
  • Like
Reactions: 2 users
QUESTION #4

What do you look for when evaluating applicants? Do you have a particular method of going over an application?

I'm going to answer a slightly different, but related question: What parts of an ERAS application are weighted highest by IM residency programs?

There is no single formula for interviews / rankings. The answer differs for US allopathic, US DO, and IMG grads. Still, some generalizations are possible:

For US Allopathic Grads:
  1. Dean's Letter -- this is probably given the most weight. Most US med schools have a standardized letter that accurately summarizes a student's performance compared with their peers. In the dean's letter, performance on Medicine and Surgery is probably weighted most highly, followed by the other third year clerkships. The quality of the last graduate of that school in our program also plays a role here. Completely unfair, I know.
  2. USMLE scores -- the great equalizer, as they are standardized across the country.
  3. An LOR from someone known to the PD/Program -- a letter from another IM program director, a recent graduate of a program, or someone known to the program is a huge bonus.
  4. Department letter -- most schools will have a "department of medicine" letter or "chair" letter. The quality of these varies. A good quality letter describes your performance in comparison to your peers. A poor quality letter simply rehashes what is already in the Dean's letter, and does not add anything to your application (doesn't hurt it, though)

Less important but helpful:
  1. Volunteer -- everyone has volunteer experience now, either because med schools seem to have added it to their curriculum or because students seem to think it's necessary to get a good spot. I use volunteer activities as a discussion item in an interview.
  2. Research -- For the absolute top tier programs, probably a must. For most others, it's nice but not vital. I have seen too many students who excel in the lab but flounder on the wards.
  3. Personal statements -- used to help ask interview questions, and make sure that you can spell, but not usually used to decide on who gets an interview or not.

And my favorite, "useless" items in your application:
  1. Letters of Rec -- I have read thousands of LOR's. Almost all state that the student is in the top 5% of students with whom they have worked. Reminds me of Lake Wobegon. Other than letters from someone I know, no value at all in my experience. Rarely we get a true gem of a letter, with statements like "seemed more interested in getting coffee than taking care of patients." or "confrontational style seemed better suited to a boardroom than the wards". Needless to say, you don't want me to remember your letters.
  2. Dubious publications -- one applicant last year listed every page on their own personal website as a publication.

For DO candidates, the Dean's letter is usually useless, because many (but not all) osteopathic schools do not include any type of comparison data for students -- i.e. they tell me you got an 87% in Medicine, but give me no sense of what that means (could be the best score in the class, or the worst). Therefore, COMLEX and USMLE scores become more vital, as do any rotations in an allopathic program. USMLE scores are a real plus, although many programs are looking at COMLEX scores now.

For IMG candidates, the Dean's letter is completely useless. USMLE scores are almost always used as a first cut. US clinical experience is highly valued, and an LOR from a US experience is vital. Observerships probably have some value, but much less so than a true rotation with primary responsibility for patient care. Visa status should have no effect by law, but probably does -- not needing a visa at all is probably a plus.
 
Last edited:
  • Like
Reactions: 3 users
QUESTION #5

Some students become paranoid over what they post publicly online on websites such as Facebook, MySpace, and even SDN. Do you ever use any outside resources when evaluating an applicant? Do you ever Google a student or look through his blog?

Usually not.

If a student refers to a blog/website/etc in their personal statement or on their application, then yes, I take a look. Seems only fair, doesn't it?

SDN allows people to be anonymous, and unless you put enough detail into your posts there's no way to figure out whom is whom.

I don't think there would be much on MySpace or FaceBook that would help me select (or deselect) candidates.

Some programs do perform a "background check" on candidates, once they've matched. There are threads about this, if you care to search for them.
 
  • Like
Reactions: 1 users
QUESTION #6

I had a few questions. Our school recommends that we have our ERAS applications completed/submitted by early September, although they say our Dean's letters aren't released until November 1st. But many applicants start getting interviews in October. For IM programs in general, do they wait until they have the Dean's letter before they start handing out interviews? Or will some programs start giving out interview spots in October (I'm guessing mostly based on Step 1 scores)?

Also, if an applicant has an average Step 1 score, will programs consider Step 2 scores if they're higher? Or will some programs simply cut applicants based on Step 1 scores?

One final quesiton about away rotations - do programs mostly interview students who do away rotations at their programs? As a West Coast student interested in some NYC/Boston programs, it's quite difficult finding the money to afford travel/living for a month to do a SubI. Does that decrease the chances of being considered for an interview by a program?

Thank you for your responses.

  1. Application timing - Many program start selecting applicants for interviews before Dean's Letters are released. Remember that we get your USMLE scores and your transcript, the latter of which gives us a good sense of how well you've done. If all looks good, many programs will offer an interview. A few programs have all P/F transcripts -- usually these are "top notch" schools, and in these cases good step scores will usually generate an interview offer prior to the Dean's letter. If there's an "anomaly" on your transcript, we will often wait for the Dean's letter to clarify.

    The most competitive programs wait for Dean's letters before offering any interviews.

    Remember that programs will get your Dean's Letter before you actually interview, and will use it to rank.

    Does applying early help get more interviews? Mostly, no. The most competitive programs review all of their applications once, and send out their interview requests. That process happens after Nov 1st. So, as long as your application is complete on Nov 1st, your chances are equal to everyone elses.

    Most other programs use a rolling process -- once your application is complete (except for the Dean's Letter) it is reviewed. Hence, if you get your application complete in Spetember, you might get an interview early. This has a few benefits -- first, having a bunch of interviews tends to make people worry less about the process, and second the early applicants tend to get first choice for interview days.

    Once the season gets late, after Nov 15th and certainly after Dec 1st, then people start to wonder why your application is so late.
  2. Step 1 vs Step 2 scores - Only the most selective programs use an "average" Step 1 score as a cut point. I personally find step 2 scores much more predictive of future performance than Step 1 scores. A high step 1 score followed by a low Step 2 score is a cause for concern.
  3. Away Rotations - Away rotations are a two edged sword. If you do really well and make a good impression, then they clearly can help you. If you don't, then they can hurt you. Most people matching into a program have not had an away rotation there, so it's clearly not a prerequisite. If you are coming from a less "prestigious" school (or from a DO school), then an away rotation, assuming you do well, can help tremendously. If you are at a top ranked med school, they are much less helpful.

All of this is for US grads. DO's and especially IMG's need to get complete applications in as early as possible.
 
  • Like
Reactions: 1 user
QUESTION #7

Can you offer any advice regarding transferring after your PGY1 year? When would be the best time to start exploring if other programs have vacancies and how would you approach the process. Also, does it have an impact on your fellowship application? I imagine you probably would have to delay it for competitive fields such as GI or cards. I'm going to be a PGY1 at a fairly competitive, good reputation program but am unsatisfied with the location- very far from home.

Yes, I know its my fault, yes I know that I still haven't started the program yet. Still I'm curious, should I decide to transfer- preferably to a program with a comparable reputation. Thanks for the help.

First, take a look at this post. This answers much of the when/how issue with PGY-2/3 transfers.

As far as fellowship is concerned, you are correct that transferring often requires delaying application for 1 year, mainly because it is difficult to establish a good research project, develop good LOR's, and manage the application process when transferring between the PGY-1 and 2 years. That being said, it's possible if you try hard enough -- get a good research project done during PGY-1, get someone from GI/Cards/etc to write you a good letter, do an outstanding job in your new PGY-2, and anything is possible.
 
  • Like
Reactions: 1 user
Question #8

I'm a MSI trying to get oriented. Can you please outline the the academic path to a specialization? If I am interested in endocrinology, would I apply to internal medicine residency and then specialize? If I am interested in pediatric endocrinology would I apply to pediatric residency first? Thank you for the great advice!

The short answer:
Yes, you first need to complete a core residency, and then during (or after) that residency you apply for a subspecialty fellowship.

The Long Answer:
"Board Certification" is an official recognition of training in either a core field (Medicine, Surgery, Peds, OB, Anesthesia, Psych, FP, etc) or a subspecialty. The whole process is overseen by the American Board of Medical Specialties (ABMS). The ABMS is composed of many smaller member boards, one for each of the core fields in medicine. Subspecialty certification is provided by each "core" medical board. Hence, the American Board of Internal Medicine certifies the subspecialties in internal medicine (Cardiology, Pulmonary, Heme/Onc, Critical Care, Rheum, Endo, etc.), and the American Board of Pediatrics certifies pediatric versions of the specialties. The ABIM specialty certifications can be viewed here (Choose Certification policies --> Subspecialty Policies from the menu on the left), and the ABP ones here (Choose "Subspecialty Certification" from the tabs at the top).

For all of these, the first step is to be accepted into a core residency. Then, during your 2nd or 3rd year or after you graduate, you apply for a specialty fellowship. In IM, most of the specialties use ERAS and the NRMP, so the process is very similar to residency applications.

There are a few "overlap" specialties. Sleep medicine, for example, is sponsored by Internal Medicine, Neurology, Psychiatry, Neuro, Family Medicine, Peds, and Otolaryngology -- hence you can become a sleep doc from any of these fields.

The process of board ceritification is very straightforward. Basically, you need to complete an approved residency or fellowship (i.e. one that is ACGME certified) and then pass an exam. Some boards require both an oral and written exam, some just written. All of the ABIM certification procedures are written only. Oh yes, I forgot to mention that you get to pay a really outrageous amount of money for this pleasure.
 
Question #9

Do you have any advice for a new starting intern?

I don't have a list of the medical stuff you will need to learn. There is SO much to learn in intern year, there is no way to make a list. The Washington Manual is a great resource to start with, for the medical issues. The real magic in intern year is getting a sense of whom is sick and who isn't, and that ain't in any book.

I do have a list of the common "non medical" intern issues:
  1. Call the attending when a patient dies at 3AM, even if you don't want to wake them up --- imagine what happens when you forget to call at 6AM, and the faculty member bumps into the family at 7:30 and asks how things are going. Not good. Always call. Always. UNLESS it is an expected death AND you are told not to call (corollary -- when someone is CMO and death is expected, ask if you need to call the attending at signout)
  2. Read on a regular basis. There is so much to learn, you must read every day. This includes day's off. Read 60 minutes each day, you will be much happier (and smarter). (You do not need to read on vacation)
  3. Find something fun to learn with each patient. No matter how "boring" the admission is, there is always something to learn. If you keep this attitude, it will help.
  4. When on call, get your team to stop working, get something to eat, and relax for 30 minutes before midnight. You may get to bed 30 minutes later, but your team will function much better.
  5. For those in a university program, don't forget what it was like to be a medical student, and treat them with respect.
  6. If you have a spouse/SO, call them when you are on call, preferably before they go to bed. Set the alarm on your pager to remind you. A 5 minute phone call when you are on call goes a long way.
  7. When you don't know something, didn't check it, or forgot -- admit it. Never, ever, ever, ever say it was "normal" or "fine". This is the cardinal sin during internship. If you do this and are caught (and trust me, you will be caught somewhere along the line), your trust/reliability will be blown and it is very difficult to get it back. High school, college, and medical school are all about hiding what you don't know. Internship is all about showing those you are working with what you don't know so they can teach you. This is very difficult since you are out of practice.
  8. Impaired physicians are a VERY common problem. Substance abuse is very common. Next year, if you don't notice an impaired physician around you, you have missed one. Chances are, someone in your training program will become impaired, either due to substances, sleep deprivation, emotional stress, etc. Always, always, always do something about it if you THINK someone might be in trouble. Never figure that someone else will do something, or that you're "not sure it's a real problem", or think that you don't have enough evidence. If you are concerned, bring it to someone's attention. be anonymous if you want to, drop off an unsigned letter to the GME office, the program director, or someone you trust.

    If you feel you might be impaired, get help. Rule of thumb: if you think you might be impaired, then you are. It is much better to get help before something bad happens.
 
  • Like
Reactions: 1 users
Question #10
Hi,

I am an international medical graduate and want to apply for a residency in IM. I gave my USMLE step 1 two years back and could not pass it (death in family four days before exam)

I came to US to do my MPH and now after completing it am again planning to give it this year. By the time I would apply i.e. next year, I would have 5-6 papers published (right now I have 2 papers and 1 chapter in Neurology).

Can you please tell me that how much impact will my previous step make in my application and also that what can I do to make my application stronger.

I am really worried......please help

I'll answer this question a bit more generally -- how important are Step 1 vs Step 2 scores?

My personal opinion and experience is that Step 2 scores are much more predictive of success in internship and residency, and hence I personally put much more weight on them. Needless to say, if you are trying to get into an uber-top residency program, then both are critical. A poor Step 1 score followed by a good step 2 score is not a huge red flag in my book. A good step 1 score followed by a low Step 2 score is a real concern.

As far as IMG vs USMG's are concerned, IMG's are much more dependent on USMLE scores as I do not have much experience with many IMG schools. I expect that many programs, myself included, screen out IMG's with low Step 1 or Step 2 scores. I get literally 800+ IMG applications each year, and I have to start screening somewhere.

However, I'll tell you a little secret. The filter system in ERAS uses your last USMLE score to filter against. Hence, if you were to retake Step 1 and do really well, you would not be filtered out and I would look at your application. I might decline to interview you once I see that fail on Step 1, or I might not. Depends on what Step 2 was. If you haven;t taken Step 2, you need to -- most programs will not look at IMG's who are not ECFMG certified.
 
Question #11
I realize it's a little early to be inquiring about IM residencies, since I'll be entering med school this fall, but I'm potentially interested in heme/onc or in being a hospitalist. I was curious as to how the "quality of med schools" are ascertained in the IM residency process, and how much of an advantage it actually provides. Is it only the top 5, or top 10, or top 20-ranked schools in which reputation makes a difference? Or is it personal knowledge of residents from that school currently in your program? From the internal medicine forums, it seems as though if you go to a very prestigious school, you can simply walk in the door of any residency program you like, even with a subpar performance, assuming there aren't major red flags on your app and you're not completely antisocial. Please feel free to correct this impression if it's wrong.

I had a shot at attending a top-5 school (at least according to USNews ranking), but turned it down to attend a school currently in the top 15, for reasons of family/friends and location. I'm really not the type of person to get too hung up on rankings or prestige, and I think I'll be happier at my chosen school, but in browsing the IM forums, med school reputation is cited as a significant factor in IM residencies. It's causing me to doubt the wisdom of my decision. Assuming my performance is roughly equal at both schools, and I'm in the middle of the pack, would I have fared better in the match from the top 5 than the school I'll soon be attending (which is also quite reputable, too)? Or are all schools in the top 20 or so gauged as "reputable" schools, and that's the end of the issue? I feel like I'm splitting hairs by overthinking this, and obviously there's nothing I can do about it now, but I do want to be aware of all of the ramifications of my school choice, both good and bad.

I also would like to know, does "unique" non-medically related activities help in getting a competitive IM residency, or do those factors not make a difference? One of the reasons that I chose my school is that by virtue of location, I'm hoping to stay involved in the performing arts in the spare time I have during med school. It's a little overambitious, I know, but I'm passionate about theater and eager to learn more about filmmaking. Most likely, I'll continue pursuing those hobbies in med school, regardless of whether they help or hinder my residency application, but it is helpful to know either way.

I do apologize for the pedestrian nature of my questions. Thank you so much for taking the time to respond. I really appreciate it.

First of all, it is NOT true that any performance at a top medical school guarantees you a spot anywhere. I have failed to interview / failed to rank many poor graduates of top medical schools. Honestly, the med school admission system is really not very good (although I'm not sure I could do better), and some people get into medical school who just don't have what it takes.

If you attend an allopathic medical school in the US, do well on your USMLE's and clinical rotations, you will be able to get a good residency, and from there a fellowship. Top 5 vs Top 15 is splitting hairs -- if your performance is good, you will get a good spot in an academic university program (assuming that's what you want). For example, the newest allopathic program is Florida state -- they graduated their first class in 2005 and seemed to have a very solid match. Yes, the better your med school is the better your chances of a residency, but your individual performance in that medical school is the driving factor most of the time.

As far as non-medical interests, I encourage you to continue to follow them if they bring you joy, as long as they do not impact your performance in medical school. They will enhance your application, but will not compensate for nor explain away other deficiencies in your record.
 
Members don't see this ad :)
Question #12

How many medicine subI's do you recommend doing? Is it okay to do just one in medicine, and another subI in another specialty we might be considering? And if we're only doing one for medicine, should it be in general medicine, or could it be a medicine subspeciality that we're interested in? (As long as it's an inpatient subspeciality rotation?)

In addition, are there 4th year electives that you think would be helpful to do before starting intern year in medicine? For example, would an ICU rotation, radiology, anesthesia (to learn intubation) etc. be useful? Or should we try to rotate through more medicine subspeciality electives like cardio, GI, etc. Thank you.

You only need to do one medicine Sub-I, and either General Med or a Subspecialty would be fine, if it's based on an inpatient service.

The answer to your second question depends on your performance in medical school. You you have done well, I highly recommend doing what you LIKE with your fourth year electives. You will learn all the clinical skills you need as an intern. In some ways, you might be better off doing things with your fourth year electives that you will not do in your internship.

If you have not done as well, then spending more time on core rotations to improve your skills will greatly help you during your internship.
 
Question #13

Thanks for your effort, you are a great resource! :) I have a few questions about fellowship application. I'm sure it varies between sub-specialty (and I'm interested in Rheumatology and Cardiolgy, which is a pretty good spread.) Anyway...

(1) When should you decide if/which subspecialty you would like to pursue?

(2) Could you give a run-down of order of importance for getting offered a position? (Come on, you knew this was coming;))

(3) Just how competitive are fellowships (say the highly-competitive cardiology)? How common is it that someone can not find a position?

Many thanks!

Your first question is somewhat rhetorical. You should decide if/which specialty when you are sure that's what you want to do. Could be anywhere from the moment you are born through being in practice for 20 years.

From a practical standpoint, it's easier if you've decided by the end of your PGY-1 or PGY-2 years. If you decide by the end of your PGY-1, you apply into the fellowship match as a PGY-2 and go directly from you residency into a fellowship. If you decide at the end of your PGY-2, you apply as a PGY-3 and then have one year off between residency and fellowship -- many people work during this year as a hospitalist.

I'll answer question #3 next:

Ultra competitive: Gastroenterology, Allergy/Immunology (esp research based programs)
Highly competitive: Cardiology, Heme/Onc
Less Competitive:pulmonary, Critical Care, Rheumatology (Becoming more competitive), General Medicine (Research based)
Not Competitive: ID, Endo, Nephrology, Geriatrics

I have seen good people not get GI and Cardiology spots. H/O is very competitive, but there are many spots out there.

Important factors for fellowship application:
  1. A great LOR from the PD
  2. A University based residency (caveat: some community based programs sponsor their own fellowships, and take applicants from their own program)
  3. A fellowship in that field at your home institution
  4. Research, especially that which generates a publication or poster at a national meeting
  5. a great LOR from a specialist personally known to the Fellowship PD
  6. Good USMLE scores will never hurt

I think that the PD LOR is probably one of the most critical pieces, along with a good research project.
 
Question #14

Dear ProgDirector - Thanks for your terrific volunteer work in this forum. I'm quite early in my career having just completed my MS-I year, but I have definitely chosen internal medicine as my specialty.

After working as a CPA in hospitals and clinics for over 20 years, I decided it was almost too late to pursue a dream of becoming a physician. I began my pre-reqs at night, studied hard, and applied with a 32 MCAT and a 4.00 GPA including several upper-division biology courses. I had a 3.95 UG GPA in accounting and a 3.8 in my master's in accounting. I had a great deal of volunteer service over many years since my employer hospital encourages volunteer and community service. In short, I think I had a pretty impressive application but I was 44 years old.

The silence was deafening - nobody was interested in even interviewing me. My state school, however, did interview me. The interview was fantastic, I was awarded a scholarship, and I finished my first year with a 4.00 GPA and was just awarded new scholarships for next year. I'm having a great time and I've learned that medicine is definitely what I'm supposed to do with the second half of my life.

My question is this: assuming I continue to do reasonably well, is age ever an issue in the residency application process? I'm sure nobody intended to discriminate against me illegally, but I think some adcom members just aren't comfortable with applicants over 40. I have no idea whether age ever factors into the selection process for postgraduate education, formally or informally, and I would certainly appreciate hearing your thoughts on this issue. Thanks.

Legally, age should never be a criterion for selection to a residency program, medical school, or anything else for that matter (except for perhaps a retirement community, which often have minimum age limits, although exactly how that is legal is unclear to me). We actually try to screen out ages in our application process -- Residency programs have the ability to "screen" data they do not want to see until after the match. We screen birthdate, which automatically screens age also. However, we often see your birthdate on your transcript, or it becomes evident from reviewing your CV that you are an older applicant.

Will it affect you in your residency search? Sadly, since it did for medical school, it probably will for residency also. The major concern that PD's will have is your ability to take call and work overnight shifts, which often becomes more difficult as people age. I have several physicians in my practice who needed to stop taking night call, albeit at a much higher age than yours.

There ain't nothing you can do about your age. Stress your practical experience, and try to get your letter writers to stress that you had no problem with overnight call despite your age (assuming that it's true). Alternatively, consider applying to programs with little or no overnight call.

Much depends on your performance in medical school. If you rock the USMLE's, get Honors in your rotations, etc, then you will have little trouble getting a spot. if you have a poor clinical record, then people may be wary about "taking a risk" with you.
 
Question #15

Hello,
I am interested in doing a residency in internal medicine, and then taking a couple years to work, maybe internationally or with NHSC or something along those lines, and then applying to a fellowship. Is it generally more difficult to get into a fellowship if you have been out of your residency for awhile?
Thanks!

It depends on the specialty, the program, and you. In general, if you are looking at a non-competitive specialty (see above), then you will have no problem applying after a few years. If you are interested in GI or Cards, then it can be more of a problem (but it depends on what you do and whom you work with).

Some programs that are very research focused may not be impressed with your service. Some socially conscious PD's may be quite impressed.

If you were a super stellar resident, then you will likely do well no matter what you do. If you are an average / below average resident and you are applying to a competitive field, then you should probably be more conservative.

Remember that you can always apply in your PGY-3 year, and then have 12 months to do international / IHS / other service work.
 
For someone wanting to stay in academic medicine, and who is aiming at either general IM, rheumatology, or ID (which are all among the lower competitive areas in IM that you've mentioned), how important is it to go to a "top" program rather than your home institution?

For family reasons, I'd like to stay. I'm sure that the clinical education is good in our IM program. Plus, it seems like some of the benefits of staying would be knowing the faculty, knowing the hospital / computer system, and knowing the 'local customs'. But, on the other hand, it would be nice to get a fresh start somewhere else and see some differences in treatment strategies.

As for research, I know some programs have fast tracks (including my home institution). So, committing to one of these programs could be helpful in transitioning from residency/fellowship to faculty. In my research niche, there aren't a lot of people locally, but its an area of expanding interest. It seems like I could try to work hard and try to position myself for future opportunities here. But, there are institutions who are already active and productive in my research area - so it seems like it would be wise to try to train with those experts.

Your home program sounds like a university based program with it's own fellowship program. In that case, you are certainly fine. Remember that residency is very busy -- having a supportive family is very helpful.

"Knowing the system" is over-rated --> within 2 months all benefit from being at your home institution is gone. In addition, some programs informally hold their own students to a higher standard early in the year, due to their experience.

Just to be clear, "Short Tracking" or "Fast Tracking" is neither really short nor fast. It is called the Research pathway and is described on the ABIM's website.

Standard training (Rheum, for example): 3 years IM + 2 years Rheum = 5 years total
Research Pathway (Rheum): 2 years IM + 3 years research + 1 year fellowship = 6 years total

So, it's faster than doing a full residency, fellowship, and then post-doc or other research fellowship. It's longer than simply completing a residency and then a fellowship.
 
Question #16

Hi, and thank you for volunteering to field our questions.

I'll likely be applying to Medicine and Med-Peds programs in the fall, with a goal of an ID residency in mind. I'll be a fourth year with very average grades at an unranked US allopathic school.

I have two questions - on FREIDA, many programs I'm looking at are labeled "community hospital, university affiliated." For fellowship applications, are these programs seen as more equivalent to community programs or university programs?

Second, you mentioned "red flags" that may need to be discussed on the application or during the interview. Would consider any of the following to be red flags: transferring medical schools, taking a year off during medical school for research, or taking a year off before applying for residency?

Thank you!

As to the status of "community hospital, university affiliated" I think it depends on two things: 1) how "affiliated" is the univeristy hospital, and 2) does the community hospital have an ID fellowship. The affiliation can vary from "the med students rotate there" to "it's a major affiliate of the Univ program". You will need to review each site to see what it consists of. In the end, the proof is in the pudding -- ask to see where their senior residents have gone. Most programs post such information on the web.

As to red flags, transferring schools depends on whether it's a step up or a step down. For example, if you started at Harvard, got a bunch of D's, and finished at a Carribean school, that's not so good. Time off is always a bit of a red flag -- unexplained time off raises concerns, so always better to explain what you were doing.
 
Question #17
Hi there,

i read through this thread and thanks so much for your great responses.

I am going to be starting my 3rd year of med school and I'm pretty sure I will be doing internal medicine. I'm still a little naive about the whole process of applying for residency. It sounded like from your previous postings that research is almost a "must" for the more competitive subspecialties--is this true? Or is it less so depending on the type of residency program. In all honesty I have discovered over the years that I have much more interest in taking care of patients than research...

Thanks.

If you are talking about the IM based subspecialties -- Cardiology, GI, Heme/Onc, etc -- then you are putting the cart a bit before the horse. In order to subspecialize in these fields, you need to first complete a 3 year IM residency. As a third year student, you want to get into a good residency program. Research is not required for that at all -- concentrate on your third year clerkship performance and Step 2. You will need a research project in your residency to help with your application for fellowship, but that is far in your future.
 
Question #18
Hello,

Thank you very much for taking the time to answer everyone's questions!
I am an international medical graduate interested in applying for an IM residency here. I graduated 2 years back and moved here last year. Since then I have been looking for clinical rotations but it was not possible for me to get any without being ecfmg certified.Now I have passed all the USMLEs and got certified, and I got into a 2-month long rotation starting next month. I am also volunteering in research for a non-profit clinic since 3 months.
The only problem is that I could not put my full efforts into studying for my USMLE steps and got poor scores in both steps, failed in step2 CK and got only a slightly better grade on my 2nd attempt in step2. At that point in my life I could not really handle immigration and adapting to a new country very well and now I am quite stuck with the scores. I do not intend to apply in high-end hospitals but even middle-tier and low-tier community hospitals seem to screen out IMG candidates with poor scores.
What should I do in order to improve my chances for matching in IM? What would be the best option for me if I don't get matched? Please help!

Unfortunately, you cannot retake the USMLE once you pass, so you are stuck with your scores. The only exception to this is if you want a license in a state that has a time limit on the USMLE's and you have not completed all three steps in that timeframe, then you can retake the exams. Unfortunately, this is no less than seven years, so it won't help you.

Hopefully your US experience will help. Otherwise, your options are limited. You could apply to Family Medicine instead, which would allow you to practice in Primary Care but limit your choices of specialties (which will be limited anyway due to your poor scores). Or, Pathology is also relatively non-competitive.

My advice is to complete your US training, apply broadly to IM (and FM if that interests you. Do NOT apply to FM if it doesn't interest you, search for any of PandaBear's posts on this forum to see why not) this year. If you do not get a spot, you are unlikely to get a spot in the future and will need to make other plans. At that point, I would suggest doing things that might improve your application AND forward your career if you do not get a spot -- i.e. if you think you might want a career in research, then get additional research experience. If you hate research, then I wouldn't pursue it further in the hopes of getting a residency spot.

For others reading this thread, a key point: A solid score on the USMLE is vital for an IMG application. Excuses about family issues, moving to this country, immigration, different study habits, cultural differences are all simply unacceptable, and in fact suggest that you will not integrate well into a US residency and hence hurt your chances for a spot dramatically.
 
Question #19
I'm just starting my 4th year and about to pick the IM programs I want to apply to for residency. I feel a bit overwhelmed by this process - I don't really know how to tell which academic programs are the best/good/ok, etc...is there any way (other than word of mouth) to figure this out? I'm trying to get an idea of which academic programs will be "reach" programs and which will be "safety" programs in my application. Any advice on how applicants can find programs that are within their reach?

Ahhh, a question with a one word answer: Mentor!

Find someone in IM at your school who can advise you on what to do, and where to apply. Remember that you really can't hurt yourself by applying for the very best programs and seeing what happens. Many people sell themselves short.

If you don't know who to go to, I have two easy suggestions:

1. The dean of students -- hey, this is their JOB! Some schools have an advising committee chaired by the Dean, and anyone from that committee should be fine.

2. The PD of the IM program at your school (assuming you have one).

If you are at a DO school, it's a bit more complicated. You are still best starting with the Dean.

And don't feel like you have to limit yourself to a single mentor. Ask a few people what they think.
 
Question #20
Hello!I have been going through your posts and i have found them very helpful.Thank you very much for volunteering your time

I am an international medical graduate from Africa.I recently finished my steps and i am hopin my results will be ready in time for the match.I had a good result in my step2, and i am awaiting my step1 results,hoping they would be good as well. What has been of great concern to me is that i have been going thru the web pages of many programs in the new england area, as this is where i reside with my husband.They all seem to have very stiff requirements for IMG's, most asking for a year of hands on clinical experience in the US.I have tried to get this, but i havent been so lucky.I want to know if these requirements are rigid, or are exceptions made occasionally depending on the candidates profile.

My other question is -I am currently pregnant,i was wondering how this could affect my chances while appearing for an interview, and i would also like to know if it would be a good idea to write about it in my personal statement .Thanks for your help

Question A: US Experience for IMG's
Most university programs insist on some sort of recent, US/Canadian clinical experience as part of an IMG application. In general, inpatient experience is more highly valued than outpatient experience. This is often a difficult task for IMG's, as they usually find themselves in "the license squeeze". Since IMG's are not medical students, they cannot obtain experience (including malpractice insurance) as a student. Since they are not part of a training program, they cannot get a training license. Since they do not have any US experience, they cannot get a full license. Without a license, they can't get any experience. It's a Catch-22. Hence, IMG's often resort to "observerships" -- a rotation where the IMG participates in rounds but has no actual responsibility for patient care, writes no notes or orders, and does not touch patients. The value of observerships is mixed. Some PD's feel that they are useful, some think they are useless. Three things are certain: 1) a letter from your clinical experience is vital; 2) Letters from physicians who work with residents/students are much more valuable than those who do not; and 3) an observership at a residency program will be of most value at that program.

Do programs "bend their rules"? Probably, but not often.

Question B: I'm pregnant, entering the match. What do I do?

The answer to this is really identical whether you are an IMG or a USMG. First, I should mention that the law protects women who are pregnant, and theoretically no one should hold that against you. However, I can certainly imagine programs looking at you (pregnant) and someone else (not pregnant) and making ranking / hiring decisions based on that.

Pregnancy and residency is all about timing:

  1. You will deliver before residency starts -- This is the easiest situation. Basically, you have no problem. You of course need to make sure that you have a very solid child care plan in place -- there is likely to be little flexibility for snow days, sick children, etc.
  2. You will deliver right before or during residency, and will be obviously pregnant at the interview -- There will be no surprise in July, so programs should be able to arrange the schedule well in advance to address the pregnancy. Talking about it in your personal statement is a double edged sword: you might receive fewer interviews based upon it, but those programs that do interview you might be more inclined to support you.
  3. You will be pregnant at your interview, but no one will be able to tell -- this is the tough one, because usually this means you will deliver right in July-Aug. There is no rule that you have to disclose that you are pregnant, and I see no reason to. However, in this case, once you match or get a spot outside the match, you absolutely should inform your program BEFORE they make the schedule.

My advice is this:
  1. If you will be delivering 2-3 months prior to the planned start of residency, then I'm not really sure that it matters at all.
  2. Apply broadly in the New England area. You might have the most luck at some of the community programs in Boston (Cambridge Hospital, Mt Auburn, etc). There are also some good programs in CT (Norwalk, Greenwich hosp).
  3. If NYC is a possibility, then there are many, many programs there to explore.
  4. For programs that are close, I see no need to mention that you are pregnant. It has nothing to do with your clinical skills.
  5. For programs that are far, and have expensive travel, it's probably best to alert them to the fact of your pregnancy. This way, you'll know that your time and money are better spent.
 
I've read many of your previous posts, elsewhere on SDN, and have noted that you've mentioned "academic difficulty" as a danger sign for interns and residents.

Particularly for interns, it seems that our job mainly requires juggling a lot of details, and making sure we marshall all appropriate resources at the right time for our patients. Certainly, we're doctors, and we use our clinical skills to observe patients, but we also do a lot of advanced secretarial work (I'm only partially joking).

So what would constitute "academic difficulty" for an intern, beyond the performance on boards? Working slowly? Presenting badly?

The ACGME now uses six "competencies", and most broadly Academic Difficulty could represent failure to perform in any of those six areas. However, realistically, problems occur in:

1. Medical Knowledge -- There is a lot to know. Much of it you need to know without looking things up. Your knowledge needs to grow with training. If your knowledge is poor enough, it can interfere with your performance. Usually, this is evident by: being extremely slow and inefficient or performing poorly in interactive conferences (morning report / teaching rounds).

2. Patient Care -- This is different than knowledge. Patient care is applying the knowledge to real patients -- developing differentials, telling who is "sick" from "not sick", and prioritizing patient care. Someone with good medical knowledge can spout off the differential for Shortness of Breath. Someone with good Patient Care skills will be able to choose the top three contenders for any given patient. I don't expect this of a beginning intern, but do by April-May.

3. Professionalism -- impossible to define, but I have seen residents do rather horrible things. The worst is lying. My experience is that anyone who lies in an important situation likely does so chronically.

----

Communication skills comes in a distant fourth. Rarely does someone get terminated for their communication skills, but it can make for some rough training. Usually, these people get labeled as "problem residents". Nurses hate them, and at the worst so do the other residents. Often people don't see how their communication (or lack thereof) can have such a devistating effect.
 
Hi, thanks so much for providing this service! I have two questions:
1) Just how much does the reputation of my medical school factor into my competitiveness for a spot in a top 25 academic IM Residency? I had the opportunity to attend more prestigious private medical schools, but opted for my very middle of the road Texas state school due to the ridiculously cheap tuition and very good clinical training. However, thier name does not hold much Internal Medicine clout and there aren't many big names to get LoR's from.
2) I have tried THREE mentors and all any of them can talk about is my home program. I have made it fairly clear that I am not interested, but still can't seem to get unbiased advice. The new PD is a temp taking over for ours that quit suddenly, and he really has been out of the game for a while. Any advice on finding help?

1. Not much. If you do well at a US school, you'll get a good IM spot. If you wanted a top 3, then it probably matters. Top 25 is a much longer list. Do well on the USMLE's and your clinical rotations, and you'll do fine in the match.

2. This is a tough situation, and your mentors are being unfair. Of course they sell their home program -- I do that also. However, I make it clear that if you're not going to stay, then I help no matter where you want to go. Your best bet is to use FREIDA to make a list of programs to consider applying to, and then bring that list to a mentor to get some opinions. If that doesn't work, you can always PM me your stats and your list and I can weigh in.
 
I have been reading your posts and found them very helpful..thanks!
I am an IMG with a posgraduation in Ob/gyn planning to switch to IM now.

1)My USMLE scores are good (99) but i have no USCE and although i am trying to get USCE, i might not be able to get it when i apply in september
2)My clinical experience since graduation has been in ob/gyn only and most of my LORS are from ob/gyn too
i was wondering if this makes my application "weak" since i'm switching specialities

What could I do to maximise my chances for an interview...any advice?Thanks.

USCE = US Clinical Experience, in case any thread readers were unaware.

Your application is what it is. Your outstanding USMLE scores will certainly help. You have three years of clinical experience in your home country. You are missing US clinical experience.

Your chances of getting any IM spot in the US are reasonably good. Many community based programs take IMG's, many with no US clinical experience if they have good clinical experience in their home country. SOme of this will depend upon the country/program. If you apply broadly, you will likely obtain some interviews and stand a reasonable chance of matching.

US clinical experience would really put your application over the top. With a good US rotation (inpatient, primary hands-on care of patients) and an LOR from the rotation, you would be almost certain to get a good spot with your excellent USMLE's.
 
Hey,

Thank you very much for answering everyone's question, I have two of my own:

1) I'm a US grad in an MD school, when should I take Step 2, I'm planning on matching in 2008. My step 1 score is a 240/97, so I didn't want to 'risk' doing poorly on step 2, but then I heard that if you do it late it could work against you and some programs won't even interview you before you completed step 2 is this true?!?!?

if so I should get it done next month right? before programs send out interview invites?!?

2) I'm thinking of doing IM or FM but definetly pursuing a primary care track if I do IM, I don't want to specialize, would a community program be a better choice than an academic program?

thanks in advance,

ocean11

As an US allopathic MD grad, with that outstanding score, applying to relatively non-competitive primary care IM / FM programs, you will do fine. This assumes that you have reasonable grades, were not convicted of murder, etc.

Step 2 is up to you. I doubt it will affect your chances of applying. STep 2 is often an application requirement for IMG's because of ECFMG issues, but is not an application requirement for US grads.

Of course, you could use the fancy trick of applying for residency, taking Step 2 such that your scores come back prior to the ROL, but not automatically releasing them. Then, if it's good, you release. If not, then you do nothing.
 
I am an IMG. I wil graduate from medical SChool in december 2009. By then I would have written Step 1,2CK and 2CS. I checked a lot of Residency websites and they indicate that ECFMG Certification is a requirement for application. Considering in my case whereby my Diploma would be ready in January and maybe sending it to ECFMG and they later confirming it...I think the whole process will end somewhere in March(at most). That is about the time that the results come out. if i miss out that would mean waiting another 1 and half years be4 starting residency and doesn't look so good.
My question: With good step 1 and 2 results pending emission of my ECFMG certificate(maybe I could provide declaration from my University that I have graduated) could I apply for residency, go on interview and match be4 the Certification???
Thanks

Some programs require ECFMG certification for application. If that is the case, then there is probably nothing you can do about it -- if they demand ECFMG certification then they will screen out your application. You can always ask.

However, many programs allow you to apply with your ECFMG pending. As long as all of your steps are completed, you will be able to participate in the match (if the only thing holding up your certification is your diploma). The fact that you are "off cycle" and will graduate in January will be a positive -- you can't get a visa until your are ECFMG certified, and most programs that want pre-certified applicants are simply tired of dealing with late visas and late residents. Perhaps this will convince some "Certified only" programs into taking a look at your application.
 
I'm an IMG who graduated in the last quarter of 2002. I started preparing for the steps two weeks after. Unfortunately, sometime into my studies, my family was struck with tragedy. This forced me to stay home since my parents are old and my brother was already in the US.

Taking care of these domestic problems required a significant input of time, which is one of the reasons I didnt pursue a residency in my home country. A second was that I was afraid a residency in IM (which is what I wish to pursue) might undermine my step scores, particularly step 2. I ended up taking a job at a primary care clinic, where I also did some social work. I cant say it was a great learning experience, but it helped me keep in touch with medicine.

I dont know if I made the right choices. At that time, I did feel they were right.

Since starting to study for the steps, I've quit my job and started doing voluntary research with a Cardiologist, who happens to be American trained. I work with him 2-3 days a week; the rest of the days, I study. I'm currently debating working on another research project with some other doctor, or partaking in social work 1-2 days a week. What do you suggest? As far as my interest level is concerned, I'm attracted to both research and social work, so it doesnt make a difference.

I'm fairly confident I'll beat 90 on my steps, possible by a fair distance. If that does happen, what are my chances of getting an IM residency in a University based program?

The obvious weak link in my application is my time since graduation. A lot of programs have a 5 year limit - I'll be applying 6 years after. Is this 5 year rule written in stone? Or can it be circumvented, provided the rest of my application is strong? I've also come across several programs with either a 10 year limit or no limit at all. How do I go about finding out if these are university based programs?

I have USCE with two strong American LORs.

I actively partook in research during med school, and am doing the same right now.

Provided I score really well in the USMLE, say 95+ on both steps, do I have a fair chance of stealing a place in a University based IM program?

Finally, do contacts matter? Can someone who knows me get me an interview call, provided I score well?

With outstanding step scores (>95), research, and US clinical experience you stand a good chance of getting a university spot. The >5 year hiatus may be a problem, but with outstanding step scores many programs will overlook this. A current US clinical experience would really help, but may be a logistical impossibility.

FREIDA lists whether programs are Univeristy, University affiliated, or Community. "University affiliated" often means that they are a community type hospital that has medical students / residents rotate there -- some are "univeristy-like" and some are not, there is no way to easily separate them short of research.

Connections always help!

Good luck!
 
Hello aProgDirector,

I am an US Citizen IMG(Graduated 2005) with approximately 11 years experience as a Lisenced/Certified Respiratory Therapist with emphasis on Critical Care and 7 years experience as an Emergency Medical Technician. I am counting on 4 strong LOR's. Looking for a 2008 IM position in New Jersey.

My questions are the following:

1) Will my US citizenship(No Visa requirement) be of any help?

2) Most importantly, will my experience be of any value? Is this type of experience considered to be USCE?

I appologize to you if you have already answered similar questions, or if these might seem trivial. All of the residents that I have spoken to tell me that yes, it should help. I would just like to have confirmation from someone on the selection side of things (ie. Program Director)...

Thank you for your time.


Will your US Citizenship be helpful? Realistically yes, although theoretically employers are not supposed to discriminate based upon nationality (although it's unclear to me whether this extends to citizenship or not). Not having to get a visa is a real plus for most programs.

Will your experience be of value? Yes, but not the way you think. It will not usually count as USCE. However, many PD's look favorably at experience in allied healthcare fields -- nursing, RT, etc. Such applicants oftne bring years of experience in areas such as interacting with patients/families, etc.

I think the real issue here is that if you graduated in 2005 and now are looking for a spot in 2008, PD's are going to want to know what you did in the meanwhile. (That's a rhetorical question, you don't need to answer it here!)

Good luck!
 
Hello,

Just wanted to thank you for your kind and helpful advice. I am a US IMG in the process of applying to internal medicine residencies in the northeast. My question is: I graduated in 6/07 and took a year off before residency so that I could optimally prepare for step 2, become ecfmg certified and pursue a certificate in public health practice (from Johns Hopkins). Do you think program directors will look poorly on this decision?

My Step 1 score is 210 and Step 2 score is 226.

Thank you in advance, it is much appreciated!

With a one year hiatus, and experience at Hopkin's public health dept, and those solid USMLE scores, lack of any visa issues, you should do well. A one year hiatus should not be a problem, especially so if you've continued some clinical experiences this year (which I assume is part of your public health work). Depends on your school and your grades of course, but if you apply broadly to a mix of university and community programs you will do fine.

Good luck!
 
Dear aProgDirector:

I'm a DO applicant planning to apply for university-based IM programs. I have a few questions to ask:

1. I actually have asked this question in another forum, but I'm very curious about what your opinion is. I hope that's ok for you. I'm very interested in going to Hem/Onc. I've written my personal statement and mostly talks about my true compassion with hem/onc. Now, since I have to go through IM first, will this hurt me? Do PDs normally expect personal statement about why choosing general internal medicine? I feel much more comfortable submitting my hem/onc PS since this is where my passion is.

2. Reading from your previous posts, sounds like getting into university-based IM programs will help tremendously to get into hem/onc fellowship. I was wondering if you could give me your opinion on what my chances are to get into univ-based IM programs? Here are my stats:
-I have done a good amount of basic science research during undergrad and med school. I only had 1 poster presentation out of it, although according to my research mentors more publications should be under way.
-USMLE Step 1 score is 227/92 and COMLEX 1 is 578/87. I took both USMLE/COMLEX Step 2 a few weeks ago and still waiting on the scores (hoping for the best).
-Top 15% in my class
-excellent extracurricular actv. such as President of a student club

3. How much of the honor society (AOA for MDs and SSP for DOs) play role in the selection process? I was qualified to be an SSP member, but I decided not to join for a personal reason. How does this hurt me?

Thanks so much for all your advice and comments.

Best,
mrdowntoearth

1. I think that a Heme/Onc PS is just fine. A large percent of univeristy based residents go on to subspecialty training. In my program, if your PS is about H/O we would try to make sure that your interviews were by some of our H/O docs. You're not interested in GIM, which is fine. You still need solid IM training to be a successful H/O doc (or so the system feels, that's an issue for an entire thread itself).

2. It's hard to answer "What are my chances?" questions. My advice to DO's looking for Univ allo spots: Consider taking the USMLE's as many Univ programs demand them (and of course, do well on them), do as well as you can in med school, and consider an away rotation at an allopathic institution early in your fourth year.

3. I doubt most allopathic PD's know what SSP is. Wouldn't lose much sleep over this.
 
i have a medical degree from Cambridge University, UK. got 99 for both usmles...but no LOR...im working here now but would want to go to USA. Is UK degrees recognised, and how should I go about getting a good clinical experience because ive already graduated and that means i cant do a clerkship...
thanks

International degrees are recognized through a process sponsored by the ECFMG. If the ECFMG certifies you, then your degree is recognized.

The clinical experience problem is one I have addressed earlier. Solutions are not ideal. In order of value:

(Least valuable)
  1. Volunteer in a physician's office who is a family member/friend
  2. Volunteer in a private physician's office.
  3. Volunteer at a community hospital (seeing inpatients)
  4. Work in a physician's office for 6+ months as a patient care tech / EKG tech / lab tech (i.e. something with direct pt contact)
  5. Work as a research fellow at a Univeristy hospital (and get something done)
  6. Work as a research fellow at a Univ hospital doing direct patient research (i.e. interacting directly with patients), and getting something done.
  7. Volunteer at a Univ hospital on an outpatient / consult service.
  8. Volunteer at a Univ hospital on an inpatient service
  9. Outpatient/Consult rotation at the level of a fourth year medical student (direct patient contact)
  10. Inpatient rotation at the level of a fourth year medical student
(Most valuable)

People may quibble about the exact ordering of this list. In addition, it's not mutally exclusive -- you could do research and volunteer on an inpatient service, etc.

How to get a volunteer / research / rotation is all about connections. I receive plenty of emails from IMG's requesting a rotation / observership / research experience -- they all go directly into the trash unless someone I know asks me to seriously consider them.
 
Hello,
My spouse has been placed in a performance plan at the end of the first year and not promoted to 2nd year. The primary reason given was that she lied to an attending that she checked her patient while she actually had not. (She had not lied, but was not able to answer exactly the attending's question as to what the urine ouput was etc.. when she had checked, so he is assuming that she lied). Also, there was mention that she had a bossy attitude etc.. (That one was from the left field).

She was already under observation at that time, as she some how offended one of the powerful attendings in the program.

For the last 6 months since this started, she has been working really hard. Got some great evals (A commendation letter from an attending). But, the PD is now saying that what you did 4 months back(lying) was a professional issue and it is out of my hands and you may be terminated by the committe.

What are our options: Should we take legal help, Should we quit before they fire us. She feels targetted and has been very brave and faced a lot.

PS: I am writing this as my wife is too distraught to do much of this and I am trying to help her.

First, take a look at a previous thread about probation and termination here.

The first issue is whether this is an academically appropriate termination, or a malicious action of a program director. From the outside, it is impossible to distinguish them. Several details suggest that this is warranted -- she was on "observation", presumably for poor performance, and then has continued to have performance issues over a reasonable period of time (6 months).

The "lying" you refer to may seem benign to an outside observer. What probably happened was something like this: The attending asked your wife what the patient's urine output was. She answered "it was OK" or "normal" or gave a volume. The attending checked the record and in fact found out that the patient had a poor (or copious) urine output. This is one of the intern cardinal sins, and is not tolerated. If she didn't know, she should have said so -- I'm sure they would have faulted her for that also, but not knowing is much better than making something up. If she confused two patients, that's understandable also but also a huge problem. Even if this is completely off the mark, the fact remains that most PD's will only terminate a resident for a pattern of performance problems -- replacing terminated residents is a huge pain.

One good evaluation from an attending does not cancel out poor evaluations. When I have a resident on a remediation/probation plan, we stress that we need to see a persisitant pattern of improvement in all clinical venues and by all evaluators to proceed.

Her current trouble is likely not related to the "lying" incident 4 months ago per se, but is likely that plus a lack of sufficient improvement over the last 4 months.

There are some inherent unfairness issues in situations like this:
  1. Residents under observation are scrutinized, and all small errors are detected. Other residents may be making similar errors, but no one notices.
  2. The alternative is a problem also: residents who are in remediation plans where the faculty are kept "blinded" often complain that they don't get enough feedback to improve. If I'm working with someone for 2 weeks and I'm not told of the issues in advance, it might take me the first week to collect enough experience to realize there is a problem and try to address it.
  3. The rumor mill starts, at all levels: faculty, residents, nurses, etc. This only worsens the situation.
  4. Some residents start to "feel the pressure to perform" which can worsen their performance.
  5. Program Directors are clearly in charge, and there is little residents can do to challenge their decisions.

What can you do?

First of all, the fact that you are here asking these questions because she is "too distraught" to do this is a huge red flag. This is her problem to solve. Your role should be in moral support.

The difference between resigning / being terminated is rather small. Any program which considers her after this will want to know the details. There may be some financial or visa issues to address, although you can usually not collect unemployment if you are terminated for cause.

Each GME program must have an internal appeal process. The PD or the director of GME should be able to explain this to her, and it's probably in the "housestaff manual" which every program has. She can appeal her decision internally, for which there really are only two arguments: 1) the termination is capricious / vindicitive / without cause; or 2) the program did not follow it's own internal rules for termination. Other than these two issues, the PD has full latitude around assessing competence.

External legal options are extremely limited, and are state dependent. You can certainly review this with a lawyer.

Finding a new program will likely be her best option. There is no easy way to do so. She could apply in the match for a new PGY-1 position (and plan to repeat her entire PGY-1). As it sounds like her current program is unwilling to promote her to a PGY-2, this may be her only option. She can of course try to get an offcycle PGY-1 spot outside the match. Her PD may be willing to help her find a new spot, and perhaps she should consider a new field. Residents who do not do well in IM/FP might consider pathology and/or psych where skills are very different.

Good luck. You have a bumpy road ahead.
 
Hi,
I'm a medical student in a Canadian medical school, and I'm interested in pursuing internal medicine residency in New York City. Do most schools differentiate Canadian applicants from American ones, or would I be treated the same way as American students?
Thanks in advance for your help!
Byronique

Most PD's will consider a Canadian medical education equivalent to a US one. You will still need ECFMG certification, and hence must have all your USMLE steps (I and II) done before the ROL due date. In addition, you'll need a visa if you are not a US citizen.
 
I did well on my IM clerkship (3 months) which I did in the summer this year. I decided I wanted to do my IM sub-I in late spring, so I can refresh my skills before starting internship (and also because I am planning to do other things the rest of the yr). This Fall, I am taking a cardio elective, as well as SICU (mainly because I want to learn procedures), however, I do have an option to switch and do a sub-i in October. On various applications I read that it is "encouraged" to have comments from MICU/sub-I on the Deans Letter. I would like to know how crucial this is since it interferes with my plans (and given that I did well in all my clinical rotations). My dean told me it doesn't matter if I want to do SICU rather than MICU, or if I want to take sub-I late since I did well in IM and have all my recommendations already, but I am not sure how representative this is of other programs. Any information would be much appreciated.

Most medical students do their Sub I early in their fourth year, to prove that they are ready for internship. Will not having one hurt you? Probably not, except perhaps for the most competitive programs. Doing it later as a "refresher" is really not necessary -- the first month of PGY-1 is a crash course in medicine.

So, overall I would suggest that you complete a Sub I early but I expect that it really won't matter much in the long run if the rest of your application looks good. If you are planning on applying to top IM programs, then an early Sub I is more important.

Addendum: A saavy SDN member notes:
Most Canadian medical schools are actually LCME-accredited, so the ECFMG certificate should not be needed. I may be wrong.

You are correct, and I stand corrected!
 
Hello!And Congratulations on your "promotion" to Asst. Moderator
Thank you for your answer to my previous questions, i found them quite helpful.

I will start this question by making a reference to your previous answer to my question

"However, I can certainly imagine programs looking at you (pregnant) and someone else (not pregnant) and making ranking / hiring decisions based on that."

I have an interview in a few days time, in a very good community program.I was hoping to go for the interview, after i have my baby [i am due about 2 weeks from now] but the most favorable date i was offered is this week.I asked for other dates but was told those dates could not be guaranteed

My question is
1 Since i am obviously heavily pregnant [9 months +] i am quite sure some1 would notice this at the interview, i am wondering if i should talk about it myself during the interview, even if a question is not raised about it, because like you said,ranking decisions might be made on it,even though they shouldnt.

Do i tell the program,a sign of how much i am interested in the program is the fact that i came for the interview even though i am heavily preg

And do i assure them of commitment to my duties, as much as any other resident,letting them know that i would be an asset to the program and not a burden

Basically what i am tryin to ask is that, should i give adequate reasons for me to be ranked considerably,even though i am heavily pregnant.[turning it into my strength]

Thank you for your help

If you are 9+ months pregnant now, PD's will really not care at all. By the time July 1st rolls around, you will have delivered, finished any maternity leave, and be ready to start work. The only problem is when you are due in May / June / July which makes it more difficult to arrange your schedule. Even with a late June pregnancy, it's not really much of a problem as long as there is sufficient notice.

So, I wouldn't worry about this at all, as long as it is safe for you to be traveling this far into your pregnancy, and as long as you feel OK to interview.

Good luck!
 
aProgDirecter

Thank you for all your comments on this forum; they have been very helpful through the residency application process. I am a MS4 at a US allopathic school applying to IM programs right now.

I hope to land a spot in a university program, with the intention of pursuing a fellowship down the road. Do you suggest I make an appoint with the IM Residency PD before interviews to get to know him/show my interest in my home program?

If so, what kinds of things about myself should I share with him or advice should I ask for?

Thanks for any help you can offer!

I guess it depends on how friendly your local IM PD is :laugh:!

In general, this is a good idea. I personally offer to meet with all of the medical students interested in IM at our local school, and I stress to them that I am willing to help them whether or not they stay. Many PD's know each other, especially those that are geographically close. Having your local PD know you might help you in getting into other programs also.

As for what to say / do / talk about --> be yourself.
 
If you're really interested in a certain program AND have not yet gotten an interview (and let's say your application, while competitive, isn't AS competitive as what may be needed to get a spot), is it worth contacting the PD by email beforehand to express your interest? Or does that come across as annoying/cloying? Would that at all improve your chances of interviewing there?

Thanks very much, by the way, for your awesome mentorship in this forum.

It is still very early in the process. It's quite possible that your dream program has not offered anyone an interview yet.

In general, I expect emails like this make little difference. I have gotten so used to people telling me that they are ranking me #1, and then not, that I simply don't believe it any more.

I doubt a short, succinct email will hurt. Keep it short -- I'd say you have 3-4 sentences.
 
Hi, read your answers to the various questions with great interest. I'd be grateful if you could tell me if prog directors for IM posts place any value on UK experience esp with post grad qualifications like MRCP and MRCGP?
Can these help reduce time off residency training?

Secondly, I qualified in 2000 from the UK and have experience in both internal and community medicine, will time since graduation count against me, esp as some progs have cut off for taking grads greater than x number of years?

Thanks for your answers.

PS: interested in Cardiology as subspeciality, how easy is it for IMGs to get in?

What value will MRCP / MRCGP have?

As usual, the official answer is "it depends". Having these certifications from the UK strongly supports your clinical skills. However, some programs tend to shy away from candidates with these advanced backgrounds -- a large part of running a residency program is maintaining a collegial, group atmosphere for the class. Having one person with tremendous experience work next to (and in the same job as) someone with almost no experience can lead to some problems. Overall I would say that it will likely work in your favor.

It will not shorten your training at all. You theoretically can get credit for your PGY-1 from the ABIM for foreign training, but it can only be requested by a PD (not by you). And, if you get credit for your PGY-1, then you're not looking for a spot in the match. So, you'd need to find someone with an open PGY-2 spot who is willing to give it to you despite your lack of US training, then petition the ABIM for PGY-1 credit, and then, if approved by the ABIM, start you at a PGY-2 level. This just realistically doesn't happen.

What about graduation time?

This may be a problem. I personally review people with extended grad times due to training outside the US (esp the UK), but not all will. You can't control this, so you simply apply and see what happens. The grad cutoff is also designed to address the "comraderie" issue noted above.

What about Cardiology?

Cards is very competitive. You will need to secure a university training program, or a community program with a Cards fellowship as your best option. You should make sure that you can "be happy" if you don't get a Cards spot, since many USMG's and IMG's don't.
 
How important are pre-clinical grades when applying for a residency? Does it matter how well you performed in years 1 and 2 if you did well on your board exams?

For the most competitive programs, yes. For most of the rest, not really. If you did poorly / failed a single preclinical rotation and then passed and did OK on Step 1, no one will really care.
 
Hi aprogdirector,

Thanks so much for your help. Your posts have really been a tremendous insight for me in this process.

I have three questions:

1) What do IM PD's look for in the Dean's Letter? Are clerkship comments (esp in IM) viewed with equal importance as grades?
2) Regarding the last paragraph of the Dean's Letter (MSPE) - where you are placed in the various "code quartiles" (outstanding, excellent, very good, good)...how important is this to the evaluation of your application in internal medicine?
3) Is it at all possible to be considered at an academic, top IM program if you are in the 2nd tier rank at a top medical school (with a solid transcript, scores, good ECs), or should you limit your application accordingly?

(Note: I know "what if's" are not great questions but I wanted to get a sense of how much importance is attributed to this last paragraph. )

Thanks again for your help.

The Dean's letter usually contains three items of value:
1. The grade distributions of the 3rd year clerkships, so we can evaluate just what an "Honors" or "Pass" means.
2. Unedited comments from your clerkships. If they have been edited, thy are not of much use (since any concerns get edited out).
3. The summary statement to which you refer.

All are important, it's hard to say which is most important.

If by 2nd tier rank, you mean second from the top, you should have a very good chance at a great university program.
 
Once again, thanks for all of the advice (on this thread and within the graduate medical forums).

When interviewing at IM programs, do applicants come off as "unprepared" if they are uncertain of what they want to specialize in? I understand that we not committed to anything until we start applying to fellowships. I have a few fields that I am interested in, plus I haven't ruled out general IM. In my personal statement, I mentioned the options that I'm considering. However, when it comes time to interview, might that make me sound "unfocused"? Is it better to go in to the interview with a false sense of certainty? I expect that the answer is "no", and I think it would always be a better conversation if we're honest about the options we are considering. But, now that I have a few invites and they are asking about what sub-specialists we might want to meet with, I don't want to appear ignorant (especially because I have not had electives in either of the main IM specialties I am interested in).

I agree that being honest is the best policy. Personally, I think it is better to be unsure of your final career pathway at this point than to be "certain" you know what you want -- I have seen a handful of people in the latter category end up unhappy in their chosen pathway.

We try to align interviews with specialties when possible. Tell programs you remain uncertain what pathway you will take, and will be happy to interview with anyone. I can't imagine that will hurt you at all.
 
Hi I have a question...

I for various reasons had a tough go in my first year of residency for multiple extrinsic factors. I was placed on probation and ultimately my contract was not renewed. I transferred to another residency program that was more attune to my needs with regards to being closer to home, and I successfully completed my residency at par with the rest of my class.

My question is that when I fill out paperwork... e.g. state licensure papers, hospital credentialing papers... is there a place where I am supposed to document what happened in my tumultuous first year? Or is that water under the bridge?

Thanks

This depends state by state. Some states ask specifically if you were put on probation -- for those you clearly answer yes. Even if they don't specifically ask, most will ask you to report all of your GME training. Even your second program will likely state that you came from your first program, so I doubt this will be a secret. That being said, it will also not likely have an effect on your licensing, although there are tough states that may want to investigate and you should count on your licensing process taking longer than usual.
 
Hello ! I have asked for your opinion before and found it very helpful...thanks ! I think this may sound very trivial..but I'd really like to know your opinion

I have started getting IV calls. But there are some programs which have sent rejects and IVs ( information from forums ) ...but I have neither. Is it ok for me to call these programs regarding the status of my application or is it too early / inappropriate to ask ?

I am worried that a call might irritate them ..and work negatively..at the same time, i dont want to miss an IV slot bcos i didnt call in time..this is my dilemma.

Please let me know what you think...Thanks!


It is still early in the process. If you feel the need to contact programs, you could email them and check to make sure that your application is complete -- "Is there anything else I can send you to help with your decision".

In general, I would wait until after the first week of Nov to contact programs.
 
aProgdirector,

I don't know if this was addressed before. I apologize if it was. How do you feel about students contacting PDs of other programs personally or via email if we have yet to receive an interview from that program?

Thanks again!

It depends on when in the application cycle.

Before Nov 1st, everyone's application is "incomplete" since MSPE's are held until Nov 1st. Hence, I would wait until at least after Nov 15th or so before contacting programs. Exceptions would be 1) already received interview at another program in same city and trying to organize travel arrangements, 2) Couples matching and couple has an interview.

It probably won't hurt if you ask nicely, once. But it's still early.
 
Hello-
Thanks for all of your great advice thus far. It has been most informative.
My question:
Im a DO student applying for rather competitive IM programs. My Step 1 scores are USMLE: 257/99 COMLEX: 754/99 and Step 2 USMLE: 261/99 and COMLEX: 696/96. Im ranked 2/218 in my class with strong LOR's and mostly Pass with Honors clinical evals. Will someone in my position be as competitive as an allopathic student with similar credentials in applying to what are generally considered to be "top notch" programs? (I hope this is general enough to be useful for others as well). Im just not sure what the general attitude toward DO candidates may be at such institutions despite what I have heard. Thanks for any insight you may provide!

Some programs are DO friendly. Some are not. As crossover from the osteopathic world to the allopathic world becomes more common, more programs are becoming comfortable with DO's.

On the other hand, new DO schools seem to be opening every day, and the quality of education at some of these new schools is uncertain. That's not to say that the quality of education at new DO schools is bad, it's just that they have no track record.

My own personal experience (based on little real data) is this:

  1. Because the quality of osteopathic schools / graduates is harder for allopathic PD's to understand, more weight is placed on COMLEX/USMLE's
  2. In general, taking the USMLE in addition to the COMLEX is a large bonus, especially if you do well.
  3. Some DO schools seem to be outaptient focused. It is not surprising to see a DO application where their third year medicine rotation is performed in a private outpt office with some inpt rounding -- no residents, etc. This can be a real concern for academic allopathic programs.
  4. Programs which have taken a DO from your school in the past who has been successful are much more likely to take you.

Your own data suggests a very successful allopathic match at a top program. Will you be "as competitive" as a similarly scoring MD applicant? Probably not, but you will be competitive enough to get a great spot in the match.
 
HEllo aProgDirector,

I have a question...

I have received a total of 6 strong LOR's from physicians including 2 from ER and Critical Care Directors, a Cardiologist, and an Assoc. Prof. My question is, should I make sure that all six letters make it to each program that I have applied to, or stick to their required numbers???? My buddy says that this would be overkill, and come across as not doing my research about the program or worse yet, not following directions... Is there such a thing as overkill is this competitive process?

Please advise. I have already sent 4 LOR's to the programs that request 4, as well as those who do not specify a number. Should I continue sending away???

Thanks

Please stop sending letters. Send 4, no more. You run a real risk of annoying people.
 
Through some experiences at my school, I've recently become very interested in Internal Medicine. Reading through this thread, it seems that there is a lot of talk about "doing well" in school, and I was wondering exactly what this means, and what sorts of things are red flags.

What exactly "doing well" means has been a bit nebulous to me, even after talking with our dean of students. In college, everything seemed pretty upfront. Although there are never any guarantees, you typically always had a baseline idea of what sort of GPA you needed in order to get into your desired medical school. Now that all seems to be out the window.

I'm sure that I am not alone in my struggle with this. In undergrad, I was in the top 10% of the class. Now in medschool where everyone was in the top 10% of their undergraduate class, I find that I am just average. Since I (along with most of my classmates) have spent my entire life being taught that "average" didn't equal "doing well," this has added a lot of stress in terms of grades. Our deans tell us not to worry, and point out that from our school people in all quartiles have matched into all specialties.

I know that performance during the 3rd year rotations and USMLE I score mean a lot, but I'm curious to know from an admissions standpoint what you look for in terms of actual grades during the first two years. In my case, I'm pretty much average, but at a top 20 school. I know that the school doesn't always mean that much, but I feel like being average now is a lot different from being average in undergraduate. I'm just not sure what it means, especially in terms of the match....

In general, the "cut" is at the level of med school. You are correct that at each stage of your training until now, the level of performance of your peers increases making your own performance more average appearing.

If you are interested in the most competitive fields (Ortho, Derm, Rads, etc) then better grades are important. In the primary care fields (IM, FM, Peds, and probably OB/GYN) as long as you do not fail a rotation or have some other "red flag", you will do fine. Even if you do have a "red flag", you are very likely to match in a residency -- there are more spots than MD applicants.

2nd year grades are very far down the list of importance, IMHO.
 
Hello,

I'm currently a M2 signed up to fulfill a 2 to 3 year primary care scholarship obligation after residency. I was hoping that I'd do residency at a prestigious academic center so that in case I decided to specialize later, my residency affiliation would allow me to get a fellowship even after I had been practicing for a while.

Recently, I found out that ERAS specifically asks if you have a military or public health obligation that you will need to fulfill following residency. I never thought this would be asked and makes me wonder if that would lead to any prejudice by programs seeking to train subspecialists or those going into academic practice (not that i'm not....it's just I have to fulfill my obligation first).

what's your opinion on this matter. Any advice is appreciated. Thanks!

I think it will have no effect whatsoever on the matching process. As a PD, I really don't care how many residents go into fellowships or into GIM -- I just want my residents to be happy and to get where they want to go.

If you have a military commitment, then that raises some concerns. The major issue is whether they can deploy you during your residency. All applicants to my program have told me that they are undeployable during training, but the military seems to keep changing the rules.

Whether your post residency commitment will be a factor in getting a fellowship is another matter. For the competitive fields, it is more difficult to get a fellowship spot once out of training -- but much of that depends on what you do with your required primary care practice time. Not worth worrying about now -- a fellowship is so far in your future it's hard to predict what will be happening.
 
Status
Not open for further replies.
Top